정수 집합 에 정 의 된 함수 f (x) 만족 조건: f (1) = 2f (2) = - 2f (n + 2) = f (n + 1) - f (n), f (n) 의 값 은?

정수 집합 에 정 의 된 함수 f (x) 만족 조건: f (1) = 2f (2) = - 2f (n + 2) = f (n + 1) - f (n), f (n) 의 값 은?


f (1) = 2, f (2) = - 2, f (n + 2) = f (n + 1) - f (n + 1) - f (n) - f (3) = f (f (2) - f (2 (1) - f (1) = - 4f (4) = f (3 (f (2) - f (2) = - 2f (5) - f (4 (3) - f (3) = 2f (6) = f (5 (5) - f (5 (5) - f (4 (4) = 4 (4 (4) - f (4 (4) - f (4 (4) - f (7) - f (f (6) - f (f (6) - f (f (6) - f (f (f (f) - f (f (f (f) - 7) - f - 2, - 4, - 2, 2, 4 순환, 순환 주 기 는 6 ∴ f (2011) = f (335 × 6 + 1) = f (1...



주어진 k * 8712 + N +, 설정 함수 f: N + → N + 만족: k 이상 의 정수 n: f (n) = n - k 에 대해 서 는 대답 하고 이 유 를 제시 하 십시오.
(1) 설정 k = 1, 그 중의 한 함수 f 재 n = 1 곳 러시아 편지 수 치 는...
(2) 설정 k = 4, 그리고 n ≤ 4 시, 2 ≤ f (n) ≤ 3, 서로 다른 함수 f 의 개 수 는...
정정: (1) 설정 k = 1, 그 중 하나 인 함수 f 재 n = 1 곳 의 함수 값 은.


문제 에는 K 보다 작 거나 같은 정수 n 이 포함 되 어 있 고 그 함수 값 도 하나의 정수 여야 한다. 그러나 대응 법칙 은 제목 에 의 해 정 해진 (1) n = k = 1, 문제 에 제 시 된 조건 은 'k 보다 큰 정수 n' 이 적합 하지 않 지만 함수 값 은 하나의 정수 여야 한다. 그러므로 f (1) 의 값 은 하나의 상수 (정수) 이다. (2) k =.